Gi Disorder

You might also like

Download as docx, pdf, or txt
Download as docx, pdf, or txt
You are on page 1of 35

A patient is admitted to the hospital with a diagnosis of diarrhea with dehydration.

The nurse recognizes that increased peristalsis


resulting in diarrhea can be related to

a.. sympathetic inhibition.


b. mixing and propulsion.
c. sympathetic stimulation.
d. parasympathetic stimulation.

parasympathetic stimulation

Parasympathetic and sympathetic branches of the autonomic nervous system (ANS) innervate the GI tract. The parasympathetic
(cholinergic) system is mainly excitatory. The sympathetic (adrenergic) system is mainly inhibitory. For example, parasympathetic
stimulation increases peristalsis and sympathetic stimulation decreases it.

A patient has an elevated blood level of indirect (unconjugated) bilirubin. One cause of this finding is that

a. the gallbladder is unable to contract to release stored bile.


b. bilirubin is not being conjugated and excreted into the bile by the liver.
c. the Kupffer cells in the liver are unable to remove bilirubin from the blood.
d. there is an obstruction in the biliary tract preventing flow of bile into the small intestine.

bilirubin is not being conjugated and excreted into bile by the liver

Bilirubin is a pigment derived from the breakdown of hemoglobin and is insoluble in water. Bilirubin is bound to albumin for transport
to the liver and is referred to as unconjugated. An indirect bilirubin determination is a measurement of unconjugated bilirubin, and
the level may be elevated in hepatocellular and hemolytic conditions.

As gastric contents move into the small intestine, the bowel is normally protected from the acidity of gastric contents by the

a. inhibition of secretin release.


b. release of bicarbonate by the pancreas.
c. release of pancreatic digestive enzymes.
d. release of gastrin by the duodenal mucosa.

release of bicarbonate by the pancreas

The hormone secretin stimulates the pancreas to secrete fluid with a high concentration of bicarbonate. This alkaline secretion
enters the duodenum and neutralizes acid in the chyme.

A patient is jaundiced and her stools are clay colored (gray). This is most likely related to

a. decreased bile flow into the intestine.


b. increased production of urobilinogen.
c. increased bile and bilirubin in the blood.
d. increased production of cholecystokinin.

decreased bile flow into the intestine

Bile is produced by the hepatocytes and is stored and concentrated in the gallbladder. When bile is released from the common bile
duct, it enters the duodenum. In the intestines, bilirubin is reduced to stercobilinogen and urobilinogen by bacterial action.
Stercobilinogen accounts for the brown color of stool. Stools may be clay-colored if bile is not released from the common bile
duct into the duodenum. Jaundice may result if the bilirubin level in the blood is elevated.

An 80-year-old man states that, although he adds a lot of salt to his food, it still does not have much taste. The nurse's response is
based on the knowledge that the older adult

a. should not experience changes in taste.


b. has a loss of taste buds, especially for sweet and salt.
c. has some loss of taste but no difficulty chewing food.
d. loses the sense of taste because the ability to smell is decreased.

has a loss of taste buds, especially or sweet and salt

Older adults have decreased numbers of taste buds and a decreased sense of smell. These age-related changes diminish the
sense of taste (especially of salty and sweet substances).
When the nurse is assessing the health perception-health maintenance pattern as related to GI function, an appropriate question to
ask is

a. "What is your usual bowel elimination pattern?"


b. "What percentage of your income is spent on food?"
c. "Have you traveled to a foreign country in the last year?"
d. "Do you have diarrhea when you are under a lot of stress?"

"Have you traveled to a foreign country in the last year?"

When assessing gastrointestinal function in relation to the health perception-health management pattern, the nurse should ask
about the patient's health practices related to the GI system, such as maintenance of normal body weight, proper dental care,
adequate nutrition, and effective elimination habits.

Query the patient about recent foreign travel with possible exposure to hepatitis or parasitic infestation. Ask about risk
behaviors for hepatitis exposure. Document whether the patient has received hepatitis A and B vaccination.
Assess the patient for habits that directly affect GI functioning. The intake of alcohol in large quantities or for long periods has
detrimental effects on the stomach mucosa. Chronic alcohol exposure causes fatty infiltration of the liver and can cause damage,
leading to cirrhosis and hepatocellular carcinoma. Obtain a history of cigarette smoking. Nicotine is irritating to the GI tract mucosa.
Cigarette smoking is related to GI cancers (especially mouth and esophageal cancers), esophagitis, and ulcers. Smoking delays the
healing of ulcers.

Family history is an important component of this health pattern. About one third of cases of colorectal cancer occur in patients with a
family history. Because of the relationship 841between colorectal and breast cancer, inquire about a history of either type of cancer
in the family.

During an examination of the abdomen the nurse should

a. position the patient in the supine position with the bed flat and knees straight.
b. listen for bowel sounds in the epigastrium and all four quadrants for 2 minutes.
c. describe bowel sounds as absent if no sound is heard in a quadrant after 2 minutes.
d. use the following order of techniques: inspection, palpation, percussion, auscultation.

listen for bowel sounds in the epigastrium and all four quadrants for 2 minutes

The nurse should listen in the epigastrium and all four quadrants for bowel sounds for at least 2 minutes. The patient should be in
the supine position and should slightly flex the knees; the head of the bed should be raised slightly. During examination of the
abdomen, the nurse auscultates before performing percussion and palpation because the latter procedures may alter the bowel
sounds. Bowel sounds cannot be described as absent until no sound is heard for 5 minutes in each quadrant.

Normal physical assessment findings of the GI system are (select all that apply)

a. nonpalpable liver and spleen.


b. borborygmi in upper right quadrant.
c. tympany on percussion of the abdomen.
d. liver edge 2 to 4 cm below the costal margin.
e. finding of a firm, nodular edge on the rectal examination.

non-palpable liver and spleen


tympany on percussion of abdomen

Normal assessment findings of the gastrointestinal system include a nonpalpable liver and spleen and generalized tympany on
percussion. Normally, bowel sounds are high pitched and gurgling; loud gurgles indicate hyperperistalsis and are called borborygmi
(stomach growling). If the patient has chronic obstructive pulmonary disease, large lungs, or a low-set diaphragm, the liver may be
palpated 0.4 to 0.8 inch (1 to 2 cm) below the right costal margin. On palpation, the rectal wall should be soft and smooth and
should have no nodules.

In preparing a patient for a colonoscopy, the nurse explains that

a. a signed permit is not necessary.


b. sedation will be used during the procedure.
c. one cleansing enema is necessary for preparation.
d. light meals should be eaten for 3 days before the procedure.

Sedation will be used during the procedure


Sedation is induced during a colonoscopy. A signed consent form is necessary for a colonoscopy. The patient should follow either a
low-residue or a full liquid diet the day before the procedure until bowel cleansing begins. Bowel cleansing should follow a split-dose
regimen. The evening before the procedure the patient should drink 2 L of oral polyethylene glycol (PEG) lavage solution. The
second 2 L dose should begin 4 to 6 hours before the procedure. Some patients may receive less solution and a cleansing enema,
but this does not occur in all patient situations.

The nurse is performing an abdominal assessment for a patient. Which assessment technique by the nurse is most accurate?

a. Palpate the abdomen before auscultation.


b. Percuss the abdomen before auscultation.
c. Auscultate the abdomen before palpation.
d. Perform deep palpation before light palpation.

auscultate the abdomen before palpation

During examination of the abdomen, auscultation is done before percussion and palpation because these latter procedures may
alter the bowel sounds.

The nurse is preparing a patient for a capsule endoscopy. What should the nurse ensure is included in the preparation?

a. Ensure the patient understands the required bowel preparation.


b. Have the patient return to the procedure room for removal of the capsule.
c. Teach the patient to maintain a clear liquid diet throughout the procedure.
d. Explain to the patient that conscious sedation will be used during capsule placement.

ensure the patient understands the required bowel preparation

A capsule endoscopy study involves the patient performing a bowel prep to cleanse the bowel before swallowing the capsule. The
patient will be on a clear liquid diet for 1 to 2 days before the procedure and will remain NPO for 4 to 6 hours after swallowing the
capsule. The capsule is disposable and will pass naturally with the bowel movement, although the monitoring device will need to be
removed.

Inspection of an older patient's mouth reveals the presence of white, curd-like lesions on the patient's tongue. What does the nurse
recognize is the most likely etiology for this abnormal assessment finding?

a. Herpesvirus
b. Candida albicans
c. Vitamin deficiency
d. Irritation from ill-fitting dentures

Candida albicans

White, curd-like lesions surrounded by erythematous mucosa are associated with oral candidiasis. Herpesvirus causes benign
vesicular lesions in the mouth. Vitamin deficiencies may cause a reddened, ulcerated, swollen tongue. Irritation from ill-fitting
dentures will cause friable, edematous, painful, bleeding gingivae.

A patient is suspected of having acute pancreatitis after presenting to the emergencgy department with severe abdomivnal pain.
Which laboratory result would best indicate the presence of acute pancreatitis?

a. Gastric pH of 1.4
b. Blood glucose of 104
c. Serum amylase of 420 U/L
d. Serum potassium of 3.5 mEq/L

serum amylase of 420 U/L

Elevated serum amylase levels indicate early pancreatic dysfunction and are used to diagnose acute pancreatitis. The normal
serum amylase levels range from 28 to 85 U/L. Serum lipase levels stay elevated longer than serum amylase in acute pancreatitis.
Blood glucose, gastric pH, and potassium levels are not direct indicators of acute pancreatic dysfunction.

The nurse is performing a focused abdominal assessment of a patient who has been recently admitted. In order to palpate the
patient's liver, where should the nurse palpate the patient's abdomen?

a. Left lower quadrant


b. Left upper quadrant
c. Right lower quadrant
d. Right upper quadrant
right upper quadrant

Although the left lobe of the liver is located in the left upper quadrant of the abdomen, the bulk of the liver is located in the right
upper quadrant.

A patient was involved in a motor vehicle crash and reports an inability to have a bowel movement. What is the best response by
the nurse?

a. "You are just too nervous to eat or drink, so there is no stool."


b. "Your parasympathetic nervous system is now working to slow the GI tract."
c. "The circulation in the GI system has been increased, so less waste is removed."
d. "Your sympathetic nervous system was activated, so there is slowing of the GI tract".

"Your sympathetic nervous system was activated, so there is slowing of the GI tract"

The constipation is most likely related to the sympathetic nervous system activation from the stress related to the accident.
Sympathetic nervous system activation can decrease peristalsis. Even without oral intake for a short time, stool will be formed. The
parasympathetic system stimulates peristalsis. The circulation to the gastrointestinal system is decreased with stress.

An older adult patient reports difficulty swallowing. Which age-related change does the nurse teach the patient about?

a. Xerostomia
b. Esophageal cancer
c. Decreased taste buds
d. Thinner abdominal wall

xerostomia

Xerostomia, decreased volume of saliva, leads to dry oral mucosa and dysphagia. Esophageal cancer is not an age-related change.
Decreased taste buds and a thinner abdominal wall do not contribute to difficulty swallowing.

A patient had a gastric resection for stomach cancer. The nurse plans to teach the patient about decreased secretion of which
hormone?

a. Gastrin
b. Secretin
c. Cholecystokinin
d. Gastric inhibitory peptide

gastrin

Gastrin is the hormone activated in the stomach (and duodenal mucosa) by stomach distention that stimulates gastric acid secretion
and motility and maintains lower esophageal sphincter tone. Secretin, cholecystokinin, and gastric inhibitory peptide are all secreted
from the duodenal mucosa.

A patient with abdominal pain is being prepared for surgery to make an incision into the common bile duct to remove stones. What
procedure will the nurse prepare the patient for?

a. Colectomy
b. Cholecystectomy
c. Choledocholithotomy
d. Choledochojejunostomy

Choledocholithotomy

A choledocholithotomy is an opening into the common bile duct for the removal of stones. A colectomy is the removal of the colon.
The cholecystectomy is the removal of the gallbladder. The Choledochojejunostomy is an opening between the common bile duct
and jejunum.

A patient reports severe pain when the nurse assesses for rebound tenderness. What may this assessment finding indicate?

a. Hepatic cirrhosis
b. Hypersplenomegaly
c. Gallbladder distention
d. Peritoneal inflammation
peritoneal inflammation

When palpating for rebound tenderness, the problem area of the abdomen will produce pain and severe muscle spasm when there
is peritoneal inflammation. Hepatic cirrhosis, hypersplenomegaly, and gallbladder distention do not manifest with rebound
tenderness.

A patient is scheduled for surgery with general anesthesia in 1 hour and is observed with a moist but empty water glass in his hand.
Which assessment finding may indicate that the patient drank a glass of water?

a. Easily heard, loud gurgling in abdomen


b. High-pitched, hollow sounds in abdomen
c. Tenderness in left upper quadrant upon palpation
d. Flat abdomen without movement upon inspection

easily heard, loud gurgling in abdomen

If the patient drank water on an empty stomach, gurgling can be assessed without a stethoscope or assessed with auscultation.
High-pitched, hollow sounds are tympanic and indicate an empty cavity. A flat abdomen and tenderness do not indicate that the
patient drank a glass of water.

When caring for the patient with heart failure, the nurse knows that which gastrointestinal process is most dependent on cardiac
output and may affect the patient's nutritional status?

a. Ingestion
b. Digestion
c. Absorption
d. Elimination

absorption

Substances that interface with the absorptive surfaces of the gastrointestinal tract (primarily in the small intestine) diffuse across the
intestinal membranes into intestinal capillaries and are then carried to other parts of the body for use in energy production. The
cardiac output provides the blood flow for this absorption of nutrients t

An older adult patient is seen in the primary care provider's office for a well check complains of difficulty swallowing. What common
effect of aging should the nurse assess for as a possible cause?

a. Anosmia
b. Xerostomia
c. Hypochlorhydria
d. Salivary gland tumor

xerostomia

Xerostomia (decreased saliva production), or dry mouth, affects many older adults and may be associated with difficulty swallowing
(dysphagia). Anosmia is loss of sense of smell. Hypochlorhydria, a decrease in stomach acid, does not affect swallowing. Salivary
gland tumors are not common.

The nurse is reviewing the home medication list for a patient admitted with suspected hepatic failure. Which medication reviewed by
the nurse could cause hepatotoxicity?

a. Digoxin
b. Nitroglycerin
c. Ciprofloxacin
d. Acetaminophen

acetaminophen

Many chemicals and drugs are potentially hepatotoxic and result in significant patient harm unless monitored closely. For example,
chronic high doses of acetaminophen and nonsteroidal antiinflammatory drugs may be hepatotoxic.

The nurse is assessing a patient admitted with a possible bowel obstruction. Which assessment finding would be expected in this
patient?

a. Tympany to abdominal percussion


b. Aortic pulsation visible in epigastric region
c. High-pitched sounds on abdominal auscultation
d. Liver border palpable 1 cm below the right costal margin

high-pitched sounds on abdominal auscultation

The bowel sounds are higher pitched (rushes and tinkling) when the intestines are under tension, as in intestinal obstruction. Bowel
sounds may also be diminished or absent with an intestinal obstruction. Normal findings include aortic pulsations on inspection and
tympany with percussion, and the liver may be palpable 1 to 2 cm along the right costal margin.

A patient is scheduled for a percutaneous transhepatic cholangiography to restore biliary drainage. The nurse discusses the
patient's health history and is most concerned if the patient makes which statement?

a. "I am allergic to bee stings."


b. "My tongue swells when I eat shrimp."
c. "I have had epigastric pain for 2 months."
d. "I have a pacemaker because my heart rate was slow."

"my tongue swells when I eat shrimp"

The percutaneous transhepatic cholangiography procedure will include the use of radiopaque contrast medium. Patients allergic to
shellfish and iodine are also allergic to contrast medium.

A patient with a family history of adenomatous polyposis had a colonoscopy with removal of multiple polyps. Which signs and
symptoms should the nurse teach the patient to report immediately?

a. Fever and abdominal pain


b. Flatulence and liquid stool
c. Loudly audible bowel sounds
d. Sleepiness and abdominal cramps

fever and abdominal pain

The patient should be taught to observe for signs of rectal bleeding and peritonitis. Fever, malaise, and abdominal pain and
distention could indicate a perforated bowel with peritonitis.

The nurse should recognize that the liver performs which functions? (select all that apply)

a. Bile storage
b. Detoxification
c. Protein metabolism
d. Steroid metabolism
e. Red blood cell (RBC) destruction

detoxification
protein metabolism
steroid metabolism
red blood cell (RBC) destruction

The liver performs multiple major functions that aid in the maintenance of homeostasis. These include metabolism of proteins and
steroids as well as detoxification of drugs and metabolic waste products. The Kupffer cells of the liver participate in the breakdown
of old RBCs. The liver produces bile, but storage occurs in the gallbladder.

The nurse is assessing a group of patients to determine their risk of vitamin D deficiency. Which of the following patients has the
highest risk for vitamin D deficiency? Vs

a. A Caucasian female who is 39 weeks gestation


b. An African-American female who is breastfeeding
c. An Asian female diagnosed with hypoglycemia
d. A Hispanic female who has a BMI of 24.1

an African-American female who is breastfeeding

Vitamin D deficiency is more frequently found among persons of African heritage and has increased in prevalence, especially
among the infants of breastfeeding African-American mothers. Caucasian females do not share these risk factors. There is no
known risk of hypoglycemia and vitamin D deficiency; however, diabetes increases the risk for vitamin D deficiency. There is no
known risk of vitamin D deficiency in normal-weight females of Hispanic heritage; however, obesity is a risk factor.
The nurse is caring for a patient diagnosed with peptic ulcer disease (PUD). The patient was prescribed the proton pump inhibitor
Prevacid (lansoprazole). Which of the following supplements may be prescribed to prevent deficiency?

a. Vitamin B12
b. Vitamin C
c. Vitamin D
d. Omega-3 fatty acids

vitamin B12

Vitamin B12 deficiency can occur as a result of the reduced gastric acidity associated with use of proton pump inhibitors, and
supplementation is often warranted. Vitamin C deficiency is not a known deficiency associated with medications. Vitamin D
deficiency may occur in patients who take cholesterol medication, and this link is currently being investigated. Omega-3 fatty acids
may be used as monotherapy or in conjunction with cholesterol medication for patients with hyperlipidemia.

Appropriate approaches used by the long-term care nurse to provide education for a 73 year old who has just been diagnosed with
diabetes include which of the following? (Select all that apply)

a. Schedule a visit by another resident who is diabetic.


b. Demonstrate food choices using food photographs.
c. Avoid discussion of the patient's favorite foods.
d. Remind the patient that a lot of damage has already occurred.
e. Encourage the patient's family to participate in teaching sessions.
f. Ask the patient about past experiences with lifestyle changes.

schedule a visit by another resident who is diabetic


demonstrate food choices using food photographs
encourage the patient's family to participate in teaching sessions
ask the patient about past experiences with lifestyle changes

Strategies to promote learning in older adults include peer teaching, visual aids, family participation, and relating new learning to
past experiences. Discussion of the patient's favorite foods is needed to determine how old favorites can be adapted to the new diet.
Reminders about the damage already done will indicate that the changes are not worth the effort.

A nurse in a home setting is assessing a 79-year-old male patient's risk for malnutrition. The nurse suspects malnutrition when
reviewing which laboratory results? (Select all that apply)

a. Body mass index (BMI) of 17


b. Waist-to-hip ratio of 1.0
c. Weight loss of 6% since last month's visit
d. Prealbumin level of 16 mg/dL
e. Hematocrit level of 50%
f. Hemoglobin level of 8.2 g/dL

body mass index (BMI) of 17


weight loss of 6% since last month's visit
hemoglobin level of 8.2 g/dL

A BMI of 18.5 to 24.9 is normal, and this patient's BMI is below normal; a major weight loss is defined as more than a 2% weight
change over 1 week; and the expected hemoglobin level for a man is 14 to 18 g/dL. The patient's values may also indicate
dehydration. The expected level for prealbumin is 15 to 36 mg/dL. A hematocrit level of 50% is within normal limits.

A person of Northern heritage is at an increased risk for which of the following? (Select all that apply)

a. Vitamin C deficiency
b. Type 1 diabetes
c. Celiac disease
d. Type 2 diabetes
e. Hypertension
f. Metabolic syndrome

type 1 diabetes
celiac disease

Type 1 diabetes and Celiac disease are more common in Northern heritage. African Americans and Hispanics are at increased risk
for Type 2 diabetes, hypertension, and metabolic syndrome. Vitamin C deficiency is not a common deficiency related to heritage or
ethnicity.

A y calls the clinic and tells the nurse that her 85-year-old mother has been nauseated all day and has vomited twice. Before the
nurse hangs up and calls the HCP, she should tell woman to

a. administer antiemetic drugs and observe skin turgor.


b. give her mother sips of water and elevate the head of her bed to prevent aspiration.
c. offer her mother a high-protein liquid supplement to drink to maintain her nutritional needs.
d. offer her mother large quantities of Gatorade to decrease the risk of sodium depletion.

give her mother sips of water and to elevate the head of her bed to prevent aspiration

Excessive replacement of fluid and electrolytes may result in adverse consequences for an older person who has heart failure or
renal disease. An older adult with a decreased level of consciousness may be at high risk for aspiration of vomitus. The elderly are
particularly susceptible to the central nervous system (CNS) side effects of antiemetic drugs; these drugs may produce confusion.
Dosages should be reduced and efficacy closely evaluated. Older patients are more likely to have cardiac or renal insufficiency,
which increases their risk for life-threatening fluid and electrolyte imbalances. High-protein drinks and high-sodium liquids may be
contraindicated.

The nurse explains to the patient with Vincent's infection that treatment will include

a. tetanus vaccinations.
b. viscous lidocaine rinses.
c. amphotericin B suspension.
d. topical application of antibiotics.

topical application of antibiotics

Vincent's infection is also known as Trench Mouth and is treated with topical applications of antibiotics. Other treatments include rest
(physical and mental); avoidance of tobacco and alcoholic beverages; soft, nutritious diet; correct oral hygiene habits; and mouth
irrigations with hydrogen peroxide and saline solutions.

The nurse teaching young adults about behaviors that put them at risk for oral cancer includes

a. use of chewing gum.


b. use of perfumed lip gloss.
c. use of smokeless tobacco.
d. drinking of carbonated beverages.

use of smokeless tobacco

Oral cancer has several predisposing risks factors:


• Lip: constant overexposure to sun, ruddy and fair complexion, recurrent herpetic lesions, irritation from pipe stem, syphilis, and
immunosuppression
• Tongue: tobacco, alcohol, chronic irritation, and syphilis
• Oral cavity: poor oral hygiene, tobacco use (e.g., pipe and cigar smoking, snuff, chewing tobacco), chronic alcohol intake, chronic
irritation (e.g., jagged tooth, ill-fitting prosthesis, chemical or mechanical irritants, and human papillomavirus [HPV] infection).

Which instructions would the nurse include in a teaching plan for a patient with mild gastroesophageal reflux disease (GERD)?

a. "The best time to take an as-needed antacid is 1 to 3 hours after meals."


b. "A glass of warm milk at bedtime will decrease your discomfort at night."
c. "Do not chew gum; the excess saliva will cause you to secrete more acid."
d. "Limit your intake of foods high in protein because they take longer to digest."

"The best time to take an antacid is 1 to 3 hours after meals."

Patients who use an as-needed antacid should do so 1 to 3 hours after eating. Teach patients that the increased saliva production
associated with chewing gum will help with GERD symptoms. The patient should not eat meals within 3 hours of bedtime. Some
foods, such as red wine, decrease lower esophageal sphincter pressure and aggravate symptoms. Milk increases gastric acid
secretion. There is no need for the patient to limit protein intake.

A patient who has undergone an esophagectomy for esophageal cancer develops increasing pain, fever, and dyspnea when a full
liquid diet is started postoperatively. The nurse recognizes that these symptoms are most indicative of

a. an intolerance to the feedings.


b. extension of the tumor into the aorta.
c. leakage of fluids into the mediastinum.
d. esophageal perforation with fistula formation into the lung.

leakage of fluids into the mediastinum

After esophageal surgery, the nurse should observe the patient for signs of leakage into the mediastinum. Symptoms that indicate
leakage are pain, increased temperature, and dyspnea.

The pernicious anemia that may accompany gastritis is due to

a. chronic autoimmune destruction of cobalamin stores in the body.


b. progressive gastric atrophy from chronic breakage in the mucosal barrier and blood loss.
c. a lack of intrinsic factor normally produced by acid-secreting cells of the gastric mucosa.
d. hyperchlorhydria from an increase in acid-secreting parietal cells and degradation of RBCs.

a lack of intrinsic factor normally produced by acid-secreting cells of the gastric mucosa

Gastritis may cause a loss of parietal cells as a result of atrophy. The source of intrinsic factor is also lost; the loss of intrinsic factor,
a substance essential for the absorption of cobalamin in the terminal ileum, ultimately results in cobalamin deficiency. With time, the
body's storage of cobalamin is depleted, and a deficiency state exists. Because cobalamin is essential for the growth and maturation
of red blood cells, the lack of cobalamin results in pernicious anemia and neurologic complications.

The nurse is teaching the patient and family that peptic ulcers are

a. caused by a stressful lifestyle and other acid-producing factors such as H. pylori.


b. inherited within families and reinforced by bacterial spread of Staphylococcus aureus in childhood.
c. promoted by factors that tend to cause oversecretion of acid, such as excess dietary fats, smoking, and H. pylori.
d. promoted by a combination of factors that may result in erosion of the gastric mucosa, including certain drugs and alcohol.

promoted by a combination of factors that may result in erosion of the gastric mucosa, including certain drugs and alcohol

Peptic ulcers develop only in the presence of an acidic environment. However, an excess of hydrochloric acid (HCl) may not be
necessary for ulcer development. The back diffusion of HCl into the gastric mucosa results in cellular destruction and inflammation.
Histamine is released from the damaged mucosa, which results in vasodilation and increased capillary permeability and further
secretion of acid and pepsin. A variety of agents (certain infections, medications, and lifestyle factors) can damage the mucosal
barrier. Helicobacter pylori can alter gastric secretion and produce tissue damage, which leads to peptic ulcer disease. Ulcerogenic
drugs, such as aspirin and NSAIDs, inhibit synthesis of prostaglandins, increase gastric acid secretion, and reduce the integrity of
the mucosal barrier. Patients taking corticosteroids, anticoagulants, and selective serotonin reuptake inhibitors are at increased risk
for ulcers. High alcohol intake stimulates acid secretion and is associated with acute mucosal lesions. Coffee (caffeinated and
uncaffeinated) is a strong stimulant of gastric acid secretion.

An optimal teaching plan for an outpatient with stomach cancer receiving radiation therapy should include information about

a. cancer support groups, alopecia, and stomatitis.


b. nutrition supplements, ostomy care, and support groups.
c. prosthetic devices, wound and skin care, and grief counseling.
d. wound and skin care, nutrition, drugs, and community resources.

wound and skin care, nutrition, drugs and community resources

Radiation therapy is used as an adjuvant to surgery or for palliation in treatment of stomach cancer. The nurse's role is to provide
detailed instructions, to reassure the patient, and to ensure completion of the designated number of treatments. The nurse should
start by assessing the patient's knowledge of radiation therapy. The nurse should teach the patient about skin care, the need for
nutrition and fluid intake during therapy, and the appropriate use of antiemetic drugs.

The teaching plan for the patient being discharged after an acute episode of upper GI bleeding includes information concerning the
importance of (select all that apply)

a. limiting alcohol intake to one serving per day.


b. only taking aspirin with milk or bread products.
c. avoiding taking aspirin and drugs containing aspirin.
d. only taking drugs prescribed by the health care provider.
e. taking all drugs 1 hour before mealtime to prevent further bleeding.

avoid taking aspirin and drugs containing aspirin


only taking drugs prescribed by the HCP
Before discharge, the patient with upper gastrointestinal (GI) bleeding and the caregiver should be taught how to avoid future
bleeding episodes. Ulcer disease and drug or alcohol abuse can cause upper GI bleeding. Help make the patient and caregiver
aware of the consequences of noncompliance with drug therapy. Emphasize that no drugs (especially aspirin and nonsteroidal
antiinflammatory drugs [NSAIDs]) other than those prescribed by the HCP should be taken. Alcohol should be eliminated because it
is a source of irritation and interfere with tissue repair.

Several patients are seen at an urgent care center with symptoms of nausea, vomiting, and diarrhea that began 2 hours ago while
attending a large family reunion potluck dinner. You question the patients specifically about foods they ingested containing

a. beef.
b. meat and milk.
c. poultry and eggs.
d. home-preserved vegetables.

meat and milk

Staphylococcus aureus toxins provoke onset of symptoms (vomiting, nausea, abdominal cramping, and diarrhea) within 30 minutes
up to 7 hours. Meat, bakery products, cream fillings, salad dressings, and milk are the usual sources of these toxins from the skin
and respiratory tract of food handlers.

The nurse is caring for a patient who complains of abdominal pain and hematemesis. Which new assessment finding(s) would
indicate the patient is experiencing a decline in condition?

a. Pallor and diaphoresis


b. Ecchymotic peripheral IV site
c. Guaiac-positive diarrhea stools
d. Heart rate 90, respiratory rate 20, BP 110/60

pallor and diaphoresis

A patient with hematemesis has some degree of bleeding from an unknown source. Guaiac-positive diarrhea stools would be an
expected finding. When monitoring the patient for stability, the nurse observes for signs of hypovolemic shock such as tachycardia,
tachypnea, hypotension, altered level of consciousness, pallor, and cool and clammy skin. An ecchymotic peripheral IV site will
require assessment to determine the need for reinsertion. Access would be critical in the immediate treatment of shock, but the
ecchymotic site does not represent a decline in condition.

The nurse is caring for a postoperative patient who has just vomited yellow green liquid and reports nausea. Which action would be
an appropriate nursing intervention?

a. Offer the patient an herbal supplement such as ginseng.


b. Apply a cool washcloth to the forehead and provide mouth care.
c. Take the patient for a walk in the hallway to promote peristalsis.
d. Discontinue any medications that may cause nausea or vomiting.

apply a cool washcloth to the forehead and provide mouth care

Cleansing the face and hands with a cool washcloth and providing mouth care are appropriate comfort interventions for nausea and
vomiting. Ginseng is not used to treat postoperative nausea and vomiting. Unnecessary activity should be avoided. The patient
should rest in a quiet environment. Medications may be temporarily withheld until the acute phase is over, but the medications
should not be discontinued without consultation with the health care provider.

Which patient would be at highest risk for developing oral candidiasis?

a. A 74-yr-old patient who has vitamin B and C deficiencies


b. A 22-yr-old patient who smokes 2 packs of cigarettes per day
c. A 32-yr-old patient who is receiving ciprofloxacin for 3 weeks
d. A 58-yr-old patient who is receiving amphotericin B for 2 days

32 y/o receiving ciprofloxacin for 3 weeks

Oral candidiasis is caused by prolonged antibiotic treatment (e.g., ciprofloxacin) or high doses of corticosteroids. Amphotericin B is
used to treat candidiasis. Vitamin B and C deficiencies may lead to Vincent's infection. Use of tobacco products leads to stomatitis,
not candidiasis.

A 74-yr-old female patient with osteoporosis is diagnosed with gastroesophageal reflux disease (GERD). Which over-the-counter
medication to treat GERD should be used with caution?
a. Sucralfate
b. Cimetidine
c. Omeprazole
d. Metoclopramide

omeprazole

There is a potential link between proton pump inhibitors (PPIs) (e.g., omeprazole) use and bone metabolism. Long-term use or high
doses of PPIs may increase the risk of fractures of the hip, wrist, and spine.

The nurse teaches senior citizens at a community center how to prevent food poisoning at social events. Which community member
statement reflects accurate understanding?

a. "Pasteurized juices and milk are safe to drink."


b. "Alfalfa sprouts are safe if rinsed before eating."
c. "Fresh fruits do not need to be washed before eating."
d. "Ground beef is safe to eat if cooked until it is brown."

"pasteurized juices and milk are safe to drink"

Drink only pasteurized milk, juice, or cider. Ground beef should be cooked thoroughly. Browned meat can still harbor live bacteria.
Cook ground beef until a thermometer reads at least 160° F. If a thermometer is unavailable, decrease the risk of illness by cooking
the ground beef until there is no pink color in the middle. Fruits and vegetables should be washed thoroughly, especially those that
will not be cooked. Persons who are immunocompromised or older should avoid eating alfalfa sprouts until the safety of the sprouts
can be ensured.

After administration of a dose of metoclopramide, which patient assessment finding would show the medication was effective?

a. Decreased blood pressure


b. Absence of muscle tremors
c. Relief of nausea and vomiting
d. No further episodes of diarrhea

relief of N/V

Metoclopramide is classified as a prokinetic and antiemetic medication. If it is effective, the patient's nausea and vomiting should
resolve. Metoclopramide does not affect blood pressure, muscle tremors, or diarrhea.

The patient receiving chemotherapy rings the call bell and reports the onset of nausea. The nurse should prepare an as-needed
dose of which medication?

a. Zolpidem
b. Ondansetron
c. Dexamethasone
d. Morphine sulfate

ondansetron

Ondansetron is a 5-HT3 receptor antagonist antiemetic that is especially effective in reducing cancer chemotherapy-induced nausea
and vomiting. Morphine sulfate may cause nausea and vomiting. Zolpidem does not relieve nausea and vomiting. Dexamethasone
is usually used in combination with ondansetron for acute and chemotherapy-induced emesis.

The patient who is admitted with a diagnosis of diverticulitis and a history of irritable bowel disease and gastroesophageal reflux
disease (GERD) has received a dose of Mylanta 30 mL PO. The nurse will determine the medication was effective when which
symptom has been resolved?

a. Diarrhea
b. Heartburn
c. Constipation
d. Lower abdominal pain

heartburn

Mylanta is an antacid that contains both aluminum and magnesium. It is indicated for the relief of gastrointestinal discomfort, such
as heartburn associated with GERD. Mylanta can cause both diarrhea and constipation as a side effect. Mylanta does not affect
lower abdominal pain.
A patient complaining of nausea receives a dose of metoclopramide. Which potential adverse effect should the nurse tell the patient
to report?

a. Tremors
b. Constipation
c. Double vision
d. Numbness in fingers and toes

tremors

Extrapyramidal side effects, including tremors and tardive dyskinesias, may occur with metoclopramide administration. Constipation,
double vision, and numbness in fingers and toes are not adverse effects of metoclopramide.

Gvv administering a dose of promethazine to a patient with nausea and vomiting, what medication side effect does the nurse explain
is common and expected?

a. Tinnitus
b. Drowsiness
c. Reduced hearing
d. Sensation of falling

drowsiness

Although being given to this patient as an antiemetic, promethazine also has sedative and amnesic properties. For this reason, the
patient is likely to experience drowsiness as an adverse effect of the medication. Tinnitus, reduced hearing, and loss of balance are
not side effects of promethazine.

The nurse is caring for a patient treated with IV fluid therapy for severe vomiting. As the patient recovers and begins to tolerate oral
intake, which food choice would be most appropriate?

a. Iced tea
b. Dry toast
c. Hot coffee
d. Plain yogurt

dry toast

Dry toast or crackers may alleviate the feeling of nausea and prevent further vomiting. Water is the initial fluid of choice. Extremely
hot or cold liquids and fatty foods are generally not well tolerated.

The nurse determines a patient has experienced the beneficial effects of therapy with famotidine when which symptom is relieved?

a. Nausea
b. Belching
c. Epigastric pain
d. Difficulty swallowing

epigastric pain

Famotidine is an H2-receptor antagonist that inhibits parietal cell output of HCl acid and minimizes damage to gastric mucosa
related to hyperacidity, thus relieving epigastric pain. It is not indicated for nausea, belching, and dysphagia.

A patient with oral cancer is not eating. A small-bore feeding tube was inserted and the patient started on enteral feedings. Which
patient goal would indicate improvement?

a. Weight gain of 1 kg in 1 week


b. Administer tube feeding at 25 mL/hr.
c. Consume 50% of clear liquid tray this shift.
d. Monitor for tube for placement and gastrointestinal residual.

weight gain of 1kg in 1 week

The goal for a patient with oral cancer that is not eating would be to note weight gain rather than loss. Consuming 50% of the clear
liquid tray is not a realistic goal. Administering feedings, monitoring tube placement, and tolerance are interventions used to achieve
the goal.
A patient with a history of peptic ulcer disease has presented to the emergency department with severe abdominal pain and a rigid,
boardlike abdomen. The health care provider suspects a perforated ulcer. Which interventions should the nurse anticipate?

a. Providing IV fluids and inserting a nasogastric (NG) tube


b. Administering oral bicarbonate and testing the patient's gastric pH level
c. Performing a fecal occult blood test and administering IV calcium gluconate
d. Starting parenteral nutrition and placing the paytient in a high-Fowler's position

providing IV fluids and inserting a NG tube

A perforated peptic ulcer requires IV replacement of fluid losses and continued gastric aspiration by NG tube. Nothing is given by
mouth, and gastric pH testing is not a priority. Calcium gluconate is not a medication directly relevant to the patient's suspected
diagnosis, and parenteral nutrition is not a priority in the short term.

The results of a patient's recent endoscopy indicate the presence of peptic ulcer disease (PUD). Which teaching point should the
nurse provide to the patient based on this new diagnosis?

a. "It would be beneficial for you to eliminate drinking alcohol."


b. "You'll need to drink at least two to three glasses of milk daily."
c. "Many people find that a minced or pureed diet eases their symptoms of PUD."
d. "Taking medication will allow you to keep your present diet while minimizing symptoms."

"It would be beneficial for you to eliminate drinking alcohol."

Alcohol increases the amount of stomach acid produced. so it should be avoided. Although there is no specific recommended
dietary modification for PUD, most patients find it necessary to make some sort of dietary modifications to minimize symptoms. Milk
may exacerbate PUD.

A patient has a sliding hiatal hernia. What nursing intervention will reduce the symptoms of heartburn and dyspepsia?

a. Keeping the patient NPO


b. Putting the bed in the Trendelenburg position
c. Having the patient eat 4 to 6 smaller meals each day
d. Giving various antacids to determine which one works for the patient

having the patient eat 4 to 6 smaller meals each day

Eating smaller meals during the day will decrease the gastric pressure and symptoms of hiatal hernia. Keeping the patient NPO or in
a Trendelenburg position is not safe or realistic for a long period of time for any patient. Varying antacids will only be done with the
health care provider's prescription, so this is not a nursing intervention.

A patient is seeking emergency care after choking on a piece of steak. The nursing assessment reveals a history of alcoholism,
cigarette smoking, and hemoptysis. Which diagnostic study is most likely to be performed on this patient?

a. Barium swallow
b. Endoscopic biopsy
c. Capsule endoscopy
d. Endoscopic ultrasonography

endoscopic biopsy

Because of this patient's history of excessive alcohol intake, smoking, and hemoptysis and the current choking episode, cancer may
be present. A biopsy is necessary to make a definitive diagnosis of carcinoma, so an endoscope will be used to obtain a biopsy and
observe other abnormalities as well. A barium swallow may show narrowing of the esophagus, but it is more diagnostic for
achalasia. An endoscopic ultrasonography may be used to stage esophageal cancer. Capsule endoscopy can show alterations in
the esophagus but is more often used for small intestine problems. A barium swallow, capsule endoscopy, and endoscopic
ultrasonography cannot provide a definitive diagnosis for cancer.

A patient was admitted with epigastric pain because of a gastric ulcer. Which patient assessment warrants an urgent change in the
nursing plan of care?

a. Back pain 3 or 4 hours after eating a meal


b. Chest pain relieved with eating or drinking water
c. Burning epigastric pain 90 minutes after breakfast
d. Rigid abdomen and vomiting following indigestion
rigid abdomen and vomiting following indigestion

A rigid abdomen with vomiting in a patient who has a gastric ulcer indicates a perforation of the ulcer, especially if the manifestations
of perforation appear suddenly. Midepigastric pain is relieved by eating, drinking water, or antacids with duodenal ulcers, not gastric
ulcers. Back pain 3 to 4 hours after a meal is more likely to occur with a duodenal ulcer. Burning epigastric pain 1 to 2 hours after a
meal is an expected manifestation of a gastric ulcer related to increased gastric secretions and does not cause an urgent change in
the nursing plan of care.

The patient with chronic gastritis is being put on a combination of medications to eradicate Helicobacter pylori. Which drugs does
the nurse know will probably be used?

a. Antibiotic(s), antacid, and corticosteroid


b. Antibiotic(s), aspirin, and antiulcer/protectant
c. Antibiotic(s), proton pump inhibitor, and bismuth
d. Antibiotic(s) and nonsteroidal antiinflammatory drugs (NSAIDs)

antibioitcs, proton pump inhibitor and bismuth

To eradicate H. pylori, a combination of antibiotics, a proton pump inhibitor, and possibly bismuth (for quadruple therapy) will be
used. Corticosteroids, aspirin, and NSAIDs are drugs that can cause gastritis and do not affect H. pylori.

The patient is having an esophagoenterostomy with anastomosis of a segment of the colon to replace the resected portion.
What initial postoperative care should the nurse expect when this patient returns to the nursing unit?

a. Turn, deep breathe, cough, and use spirometer every 4 hours.


b. Maintain an upright position for at least 2 hours after eating.
c. NG will have bloody drainage and it should not be repositioned.
d. Keep in a supine position to prevent movement of the anastomosis.

NG will have bloody drainage and it should not be repositioned

The patient will have bloody drainage from the nasogastric (NG) tube for 8 to 12 hours, and it should not be repositioned or
reinserted without contacting the surgeon. Turning and deep breathing will be done every 2 hours, and the spirometer will be used
more often than every 4 hours. Coughing would put too much pressure in the area and should not be done. Because the patient will
have the NG tube, the patient will not be eating yet. The patient should be kept in a semi-Fowler's or Fowler's position, not supine, to
prevent reflux and aspiration of secretions.

A patient who had a gastroduodenostomy (Billroth I operation) for stomach cancer reports generalized weakness, sweating,
palpitations, and dizziness 15 to 30 minutes after eating. What long-term complication does the nurse suspect is occurring?

a. Malnutrition
b. Bile reflux gastritis
c. Dumping syndrome
d. Postprandial hypoglycemia

dumping syndrome

After a Billroth I operation, dumping syndrome may occur 15 to 30 minutes after eating because of the hypertonic fluid going to the
intestine and additional fluid being drawn into the bowel. Malnutrition may ovccur but does not cause these symptoms. Bile reflux
gastritis cannot happen when the stomach has been removed. Postprandial hypoglycemia occurs with similar symptoms, but 2
hours after eating.

The nurse is teaching a group of college students how to prevent food poisoning. Which comment shows an understanding of
foodborne illness protection?

a. "Eating raw cookie dough from the package is a great snack when you do not have time to bake."
b. "Since we only have one cutting board, we can cut up chicken and salad vegetables at the same time."
c. "To save refrigerator space, leftover food can be kept on the counter if it is in sealed containers."
d. "When the cafeteria gave me a pink hamburger, I sent it back and asked for a new bun and clean plate."

"When the cafeteria gave me a pink hamburger, I sent it back and asked for a new bun and clean plate."

The student who did not accept the pink hamburger and asked for a new bun and clean plate understood that the pink meat may not
have reached 160°F and could be contaminated with bacteria. Improperly storing cooked foods, eating raw cookie dough from a
refrigerated package, and only using one cutting board without washing it with hot soapy water between the chicken and salad
vegetables could all lead to food poisoning from contamination.
A woman is 5 ft, 6 in (166 cm) tall and weighs 200 lb (90.9 kg) with a waist-to-hip ratio of 0.7. The nurse counsels the patient with
the knowledge that the patient is at greatest risk for

a. heart disease
b. osteoporosis
c. diabetes mellitus
d. endometrial cancer

osteoporosis

A patient who is obese (BMI of 32.2) but has a waist-to-hip ratio of less than 0.8, indicating gynoid (pear) obesity, has an increased
risk for osteoporosis. The other conditions are risks associated with android obesity.

Which explanation about weight reduction should be included when teaching the obese patient and her obese husband?

a. Weight gain is caused by psychologic factors.


b. Daily weighing is recommended to monitor weight loss.
c. Fat is not burned until the glycogen-water pool is depleted.
d. Men lose weight less quickly than women because they have a higher percentage of metabolically less-active fat.

fat is not burned until the glycogen-water pool is depleted

With reducing diets that severely restrict carbohydrates, the body's glycogen stores become depleted within a few days. The
glycogen normally binds to water in fat cells and it is this water loss that causes weight loss in the first few days. Fat is not burned
until the glycogen-water pool is depleted. Although psychosocial components (i.e., using food for comfort or reward and inability to
buy high- nutritional quality food) may have an influence on weight gain, these factors along with lack of physical exercise,
underestimation of portion size, and genetics contribute to weight gain. Weekly weighing is recommended as a more reliable
indicator of weight loss because daily weighing shows frequent fluctuation from retained water (including urine) and elimination of
feces. Men are able to lose weight more quickly than women because women have a higher percentage of metabolically less-active
fat.

A patient asks the nurse about taking phentermine and topiramate (Qsymia) for weight loss. To avoid side effects, it is important for
the nurse to determine whether the patient has a history of

A. Glaucoma
B. Hypertension
C. Valvular heart disease
D. Irritable bowel disease

glaucoma

Qsymia is a combination of phentermine and topiramate. It must not be used in patients with glaucoma or hyperthyroidism.

A 40-year-old severely obese female patient with type 2 diabetes wants to lose weight. After learning about the surgical procedures,
she thinks a combination of restrictive and malabsorptive surgery would be best. Which procedure should the nurse teach her
about?

A. Lipectomy
B. Roux-en-Y gastric bypass
C. Adjustable gastric banding
D. Vertical sleeve gastrectomy

Roux-en-Y gastric bypass

The Roux-en-Y gastric bypass is a common combination of restrictive (limiting the size of the stomach) and malabsorptive (less food
is absorbed) surgery. Lipectomy is used to remove unsightly flabby folds of adipose tissue. Adjustable gastric banding is the most
common restrictive procedure. Vertical sleeve gastrectomy is a restrictive procedure that preserves stomach function.

The nurse providing education to a post gastric bypass client indicates the following choices as appropriate for a Step II diet:

A. Broth, clear jello, fluids flavored with enhancers


B. Pudding, mashed potatoes, cooked cereals
C. Protein shakes, sugar free ice cream, creamed soups
D. Pureed meats, vegetables, and fruits

protein shakes, sugar free ice cream, creamed soups


Step I is a clear liquid diet and consists of broth, clear jello and fluids flavored with enhancers.

Step II is a full liquid diet and can include protein shakes, sugar free ice cream and creamed soups.

Step III consists of soft foods that are easily digestible including pudding, mashed potatoes, cooked cereals and pureed meats,
vegetables and fruit.

Which laboratory findings should the nurse expect in the patient with persistent vomiting?

a. ↓ pH, ↑ sodium, ↓ hematocrit


b. ↑ pH, ↓ chloride, ↓ hematocrit
c. ↑ pH, ↓ potassium, ↑ hematocrit
d. ↓ pH, ↓ potassium, ↑ hematocrit

↑ pH, ↓ potassium, ↑ hematocrit

The loss of gastric hydrochloric acid causes metabolic alkalosis and an increase in pH; loss of potassium, sodium, and chloride; and
loss of fluid, which increases the hematocrit.

A patient who has been vomiting for several days from an unknown cause is admitted to the hospital. What should the nurse
anticipate will be included in collaborative care?

a. Oral administration of broth and tea


b. IV replacement of fluid and electrolytes
c. Administration of parenteral antiemetics
d. Insertion of a nasogastric (NG) tube for suction

IV replacement of fluid and electrolytes

The patient with severe or persistent vomiting requires IV replacement of fluids and electrolytes until able to tolerate oral intake to
prevent serious dehydration and electrolyte imbalances. Oral fluids are not given until vomiting has been relieved and parenteral
antiemetics are often not used until a cause of the vomiting can be established. Nasogastric (NG) intubation may be indicated in
some cases but fluid and electrolyte replacement is the first priority.

How should the nurse teach the patient with a hiatal hernia or GERD to control symptoms?

a. Drink 10 to 12 oz of water with each meal.


b. Space six small meals a day between breakfast and bedtime.
c. Sleep with the head of the bed elevated on 4- to 6-inch blocks
d. Perform daily exercises of toe-touching, sit-ups, and weight lifting.

sleep with the head of the bed elevated on 4-to-6 inch blocks

The use of blocks to elevate the head of the bed facilitates gastric emptying by gravity and is strongly recommended to prevent
nighttime reflux. Liquids should be taken between meals to prevent gastric distention with meals. Small meals should be eaten
frequently but patients should not eat at bedtime or lie down for 2 to 3 hours after eating. Activities that involve increasing
intraabdominal pressure, such as bending over, lifting, or wearing tight clothing, should be avoided.

Which type of gastritis is most likely to occur in a college student who has an isolated drinking binge?

a. Acute gastritis
b. Chronic gastritis
c. Helicobacter pylori gastritis
d. Autoimmune metaplastic atrophic gastritis

acute gastritis

Acute gastritis is most likely to occur with an isolated drinking binge. Chronic gastritis is usually caused by Heliobacter pylori or viral
and fungal infections. Autoimmune gastritis is an inherited condition.

Nursing management of the patient with chronic gastritis includes teaching the patient to

a. take antacids before meals to decrease stomach acidity.


b. maintain a nonirritating diet with six small meals a day.
c. eliminate alcohol and caffeine from the diet when symptoms occur.
d. use nonsteroidal antiinflammatory drugs (NSAIDs) instead of aspirin for minor pain relief.
maintain a nonirritating diet with six small meals a day

nonirritating diet with six small meals a day is recommended to help control the symptoms of gastritis. Nonsteroidal antiinflammatory
drugs (NSAIDs) are often as irritating to the stomach as aspirin and should not be used in the patient with gastritis. Antacids are
often used for control of symptoms but have the best neutralizing effect if taken after meals. Alcohol and caffeine should be
eliminated entirely because they may precipitate gastritis.

Duodenal and gastric ulcers have similar as well as differentiating features. What are characteristics unique to duodenal ulcers
(select all that apply)?

a. Pain is relieved with eating food.


b. They have a high recurrence rate.
c. Increased gastric secretion occurs.
d. Associated with Helicobacter pylori infection.
e. Hemorrhage, perforation, and obstruction may result.
f. There is burning and cramping in the midepigastric area.

pain is relieved with eating food


increased gastric secretion ocgcurs
there is burning and cramping in the midepigastric area

Duodenal ulcers have increased gastric secretion, which causes the burning and cramping in the midepigastric area, and the pain is
relieved with food. The other options occur with both duodenal and gastric ulcers.

Corticosteroid medications are associated with the development of peptic ulcers because of which probable pathophysiologic
mechanism?

a. The enzyme urease is produced.


b. Secretion of hydrochloric acid is increased.
c. The rate of mucous cell renewal is decreased.
d. The synthesis of mucus and prostaglandins is inhibited.

rate of mucous cell renewal is decreased

Corticosteroids decrease the rate of mucous cell renewal. H. pylori produces the enzyme urease. Alcohol ingestion increases the
secretion of hydrochloric acid. Aspirin and NSAIDs inhibit the synthesis of mucus and prostaglandins.

Regardless of the precipitating factor, what causes the injury to mucosal cells in peptic ulcers?

a. Acid back diffusion into the mucosa


b. The release of histamine from GI cells
c. Ammonia formation in the mucosal wall
d. Breakdown of the gastric mucosal barrier

acid back diffusion into the mucosa

The ultimate damage to the tissues of the stomach and duodenum, precipitating ulceration, is acid back diffusion into the mucosa.
The gastric mucosal barrier is protective of the mucosa but without the acid environment and damage, ulceration does not occur.
Ammonia formation by H. pylori and release of histamine impair the barrier but are not directly responsible for tissue injury.

What is the rationale for treating acute exacerbation of peptic ulcer disease with NG intubation?

a. Stop spillage of GI contents into the peritoneal cavity


b. Remove excess fluids and undigested food from the stomach
c. Feed the patient the nutrients missing from the lack of ingestion
d. Remove stimulation for hydrochloric acid and pepsin secretion by keeping the stomach empty

remove stimulation for HCl and pepsin secretion by keeping the stomach empty

NG intubation is used with acute exacerbation of peptic ulcer disease to remove the stimulation for hydrochloric acid (HCl) and
pepsin secretion by keeping the stomach empty. Stopping the spillage of GI contents into the peritoneal cavity is used for peritonitis.
Removing
excess fluids and undigested food from the stomach is the rationale for using NG intubation for gastric outlet obstruction.

Which medications are used to decrease gastric or hydrochloric acid secretion? (select all that apply)

a. Famotidine (Pepcid)
b. Sucralfate (Carafate)
c. Omeprazole (Prilosec)
d. Misoprostol (Cytotec)
e. Amoxicillin/clarithromycin/omeprazole

famotidine
omeprazole
misoprostol

Famotidine (Pepcid) reduces HCl secretion by blocking histamine and omeprazole (Prilosec) decreases gastric acid secretion by
blocking adenosine triphosphatase (ATPase) enzyme. Sucralfate (Carafate) coats the ulcer to protect it from acid erosion.
Misoprostol (Cytotec) mixture has antisecretory effects. Amoxicillin/clarithromycin/omeprazole are used in patients with verified H.
pylori.

Thge nurse determines that teaching for the patient with peptic ulcer disease has been effective when the patient makes which
statement?

a. "I should stop all my medications if I develop any side effects."


b. "I should continue my treatment regimen as long as I have pain."
c. "I have learned some relaxation strategies that decrease my stress."
d. "I can buy whatever antacids are on sale because they all have the same effect."

"I have learned some relaxation strategies that decrease my stress."

Increased vagal stimulation from emotional stress causes hypersecretion of hydrochloric acid and stress reduction is an important
part of the patient's management of peptic ulcers, especially duodenal ulcers. If side effects to medications develop, the patient
should notify the health care provider before altering the drug regimen. Although effective treatment will promote pain relief in
several days, the treatment regimen should be continued until there is evidence that the ulcer has healed completely. Interchanging
brands and preparations of antacids and histamine (H2)- receptor blockers without checking with health care providers may cause
harmful side effects and patients should take only prescribed medications.

A patient with a history of peptic ulcer disease is hospitalized with symptoms of a perforation. During the initial assessment, what
should the nurse expect the patient to report?

a. Vomiting of bright-red blood


b. Projectile vomiting of undigested food
c. Sudden, severe upper abdominal pain and back pain
d. Hyperactive stomach sounds and upper abdominal swelling

sudden, severe upper abdominal pain and back pain

Perforation of an ulcer causes sudden, severe abdominal pain that is often referred to the back, accompanied by a rigid, boardlike
abdomen and other signs of peritonitis. Vomiting of blood indicates hemorrhage of an ulcer and gastric outlet obstruction is
characterized by projectile vomiting of undigested food, hyperactive stomach sounds, and upper abdominal swelling.

Following a Billroth II procedure, a patient develops dumping syndrome. The nurse should explain that the symptoms associated
with this problem are caused by

a. distention of the smaller stomach by too much food and fluid intake.
b. hyperglycemia caused by uncontrolled gastric emptying into the small intestine.
c. irritation of the stomach lining by reflux of bile salts because the pylorus has been removed.
d. movement of fluid into the small bowel because concentrated food and fluids move rapidly into the intestine.

movement of fluid into the small bowel because concentrated food and fluids move rapidly into the intestine

Because there is no sphincter control of food taken into the stomach following a Billroth II procedure, concentrated food and fluid
move rapidly into the small intestine, creating a hypertonic environment that pulls fluid from the bowel wall into the lumen of the
intestine, reducing plasma volume and distending the bowel. Postprandial hypoglycemia occurs when the concentrated
carbohydrate bolus in the small intestine results in hyperglycemia and the release of excessive amounts of insulin into the
circulation, resulting in symptoms of hypoglycemia. Irritation of the stomach by bile salts causes epigastric distress after meals, not
dumping syndrome.

The nurse listens to bowel sounds for 60 seconds and does not hear gurgling. Which action should the nurse take next?

a. Document the bowel sounds as hypoactive.


b. Continue to listen for at least another 60 seconds.
c. Administer the prescribed drug for constipation.
d. Review the patient's dietary intake for the past 24 hours.

continue to listen for at least another 60 seconds

Listen for bowel sounds for at least 2 minutes. If bowel sounds are not heard, note the amount of time listening in each quadrant.

The most appropriate therapy for a patient with acute diarrhea caused by a viral infection is to

a. increase fluid intake.


b. administer an antibiotic.
c. administer an antimotility drug.
d. quarantine the patient to prevent spread of the virus.

increase fluid intake

Acute diarrhea resulting from infectious causes (e.g., virus) is usually self-limiting. The major concerns are transmission prevention,
fluid and electrolyte replacement, and resolution of the diarrhea. Antidiarrheal agents are contraindicated in the treatment of
infectious diarrhea because they potentially prolong exposure to the infectious organism. Antibiotics are rarely used to treat acute
diarrhea. To prevent transmission of diarrhea caused by a virus, wash your hands before and after contact with the patient and when
handling body fluids of any kind. Flush vomitus and stool down the toilet, and wash contaminated clothing immediately with soap
and hot water.

A 35-year-old female patient is admitted to the emergency department with acute abdominal pain. Which medical diagnoses should
you consider as possible causes of her pain (select all that apply)?

a. Gastroenteritis
b. Ectopic pregnancy
c. Gastrointestinal bleeding
d. Irritable bowel syndrome
e. Inflammatory bowel disease

gastroenteritis
ectopic pregnancy
GI bleeding
IBS
IBD

All these conditions could cause acute abdominal pain.

Assessment findings suggestive of peritonitis include (select all that apply)

a. rebound tenderness.
b. a soft, distended abdomen.
c. dull, intermittent abdominal pain.
d. shallow respirations with bradypnea.
e. observing that the patient is lying still.

rebound tenderness
observing that the patient is lying still

With peritoneal irritation, the abdomen is hard, and the patient has severe continuous abdominal pain that is worse with any sudden
movement. Palpating the abdomen and releasing the hands suddenly causes sudden movement within the abdomen and severe
pain. This is called rebound tenderness. The patient lies very still and takes shallow breaths. Abdominal distention, tachypnea, fever,
and tachycardia may occur.

In planning care for the patient with Crohn's disease, the nurse recognizes that a major difference between ulcerative colitis and
Crohn's disease is that Crohn's disease

a. frequently results in toxic megacolon.


b. causes fewer nutritional deficiencies than ulcerative colitis.
c. often recurs after surgery, whereas ulcerative colitis is curable with a colectomy.
d. is manifested by rectal bleeding and anemia more often than is ulcerative colitis.

often recurs after surgery, whereas ulcerative colitis is curable with a colectomy

Ulcerative colitis affects only the colon and rectum; it can cause megacolon and rectal bleeding, but not nutrient malabsorption.
Surgical removal of the colon and rectum cures it. Crohn's disease usually involves the ileum, where bile salts and vitamin
cobalamin are absorbed. After surgical treatment, disease recurrence at the site is common.

The nurse performs a detailed assessment of the abdomen of a patient with a possible bowel obstruction, knowing that
manifestations of an obstruction in the large intestine are (select all that apply)gg

a. persistent abdominal pain.


b. marked abdominal distention.
c. diarrhea that is loose or liquid.
d. colicky, severe, intermittent pain.

persistant abdominal pain


marked abdominal distention

With lower intestinal obstructions, abdominal distention is markedly increased and pain is persistent. Onset of a large intestine
obstruction is gradual, vomiting is rare, and there is usually absolute constipation, not diarrhea.

A patient with stage I colorectal cancer is scheduled for surgery. Patient teaching for this patient would include an explanation that

a. chemotherapy will begin after the patient recovers from the surgery.
b. both chemotherapy and radiation can be used as palliative treatments.
c. follow-up colonoscopies will be needed to ensure that the cancer does not recur.
d. a wound, ostomy, and continence nurse will visit the patient to identify an abdominal site for the ostomy.

f/u colonoscopies will be needed to ensure that the cancer does not recur

Stage I colorectal cancer is treated with surgical removal of the tumor and reanastomosis, and so there is no ostomy. Chemotherapy
is not recommended for stage I tumors. Follow-up colonoscopy is recommended because colorectal cancer can recur.

The nurse determines a patient undergoing ileostomy surgery understands the procedure when the patient states

a. "I should only have to change the pouch every 4 to 7 days."


b. "The drainage in the pouch will look like my normal stools."
c. "I may not need to wear a drainage pouch if I irrigate it daily."
d. "Limiting my fluid intake should decrease the amount of output."

"I should only have to change the pouch every 4 to 7 days

Because ileostomy drainage is a liquid to thin paste, the patient will need to wear a drainage bag at all times. The patient should use
an open-ended drainable pouch. It is worn for 4 to 7 days. Output from a sigmoid colostomy resembles normally formed stool, and
some patients are able to regulate emptying time so they do not need to wear an ostomy pouch.

In contrast to diverticulitis, the patient with diverticulosis

a. has rectal bleeding.


b. often has no symptoms.
c. has localized cramping pain.
d. frequently develops peritonitis.

often has no symptoms

Many people with diverticulosis have no symptoms. Patients with diverticulitis have symptoms of inflammation. Diverticulitis can lead
to obstruction or perforation.

A nursing intervention that is most appropriate to decrease postoperative edema and pain after an inguinal herniorrhaphy is

a. applying a truss to the hernia site.


b. allowing the patient to stand to void.
c. supporting the incision during coughing.
d. applying a scrotal support with an ice bag.

applying a scrotal support with ice bag

Scrotal edema is a painful complication after an inguinal hernia repair. Scrotal support with application of an ice bag may help
relieve pain and edema.
The nurse determines that the goals of dietary teaching have been met when the patient with celiac disease selects from the menu

a. scrambled eggs and sausage.


b. buckwheat pancakes with syrup.
c. oatmeal, skim milk, and orange juice.
d. yogurt, strawberries, and rye toast with butter.

scrambled eggs and sausage

Celiac disease is treated with lifelong avoidance of dietary gluten (wheat, barley, oats, rye products). Although pure oats do not
contain gluten, oat products can become contaminated with wheat, rye, and barley during the milling process. Gluten is also found
in some medications and in many food additives, preservatives, and stabilizers.

What should a patient be taught after a hemorrhoidectomy?

a. Take mineral oil before bedtime.


b. Eat a low-fiber diet to rest the colon.
c. Administer oil-retention enema to empty the colon.
d. Use prescribed pain medication before a bowel movement.

use prescribed pain medication before a bowel movement

After a hemorrhoidectomy, the patient usually dreads the first bowel movement and often resists the urge to defecate. Give pain
medication before the bowel movement to reduce discomfort. The patient should avoid constipation and straining. A high-fiber diet
can reduce constipation. A stool softener such as docusate (Colace) is usually ordered for the first few postoperative days. If the
patient does not have a bowel movement within 2 to 3 days, an oil-retention enema is administered.

The nurse is conducting discharge teaching for a patient with metastatic lung cancer who was admitted with a bowel impaction.
Which instructions would be most helpful to prevent further episodes of constipation?

a. Maintain a high intake of fluid and fiber in the diet.


b. Discontinue intake of medications causing constipation.
c. Eat several small meals per day to maintain bowel motility.
d. Sit upright during meals to increase bowel motility by gravity.

maintain a high intake of fluid and fiber in the diet

Increased fluid intake and a high-fiber diet reduce the incidence of constipation caused by immobility, medications, and other
factors. Fluid and fiber provide bulk that in turn increases peristalsis and bowel motility. Analgesics taken for lung cancer probably
cannot be discontinued. Eating several small meals per day and position do not facilitate bowel motility.

The nurse should administer an as-needed dose of magnesium hydroxide after noting what information when reviewing a patient's
medical record

a. Abdominal pain and bloating


b. No bowel movement for 3 days
c. A decrease in appetite by 50% over 24 hours
d. Muscle tremors and other signs of hypomagnesemia

no bowel movement for 3 days

Magnesium hydroxide is an osmotic laxative that produces a soft, semisolid stool usually within 15 minutes to 3 hours. This
medication would benefit the patient who has not had a bowel movement for 3 days. It would not be given for abdominal pain and
bloating, decreased appetite, or signs of hypomagnesemia.

The nurse is preparing to administer a dose of bisacodyl to a patient with constipation and the patient asks how it will work. What is
the best response by the nurse?vgyg

a. "It will increase bulk in the stool."


b. "It will lubricate the intestinal tract to soften feces."
c. "It will increase fluid retention in the intestinal tract."
d. "It will increase peristalsis by stimulating nerves in the colon wall."

"It will increase peristalsis by stimulating nerves in the colon wall"

Bisacodyl is a stimulant laxative that aids in producing a bowel movement by irritating the colon wall and stimulating enteric nerves.
Fiber and bulk- forming drugs increase bulk in the stool. Water and stool softeners soften feces, and saline and osmotic solutions
cause fluid retention in the intestinal tract.

The nurse is preparing to administer a scheduled dose of docusate sodium when the patient reports an episode of loose stool and
does not want to take the medication. What is the appropriate action by the nurse?

a. Write an incident report about this untoward event.


b. Attempt to have the family convince the patient to take the ordered dose.
c. Withhold the medication at this time and try to administer it later in the day.
d. Chart the dose as not given on the medical record and explain in the nursing progress notes.

chart the dose as not given on the medical record and explain in the nursing progress notes

Whenever a patient refuses medication, the dose should be charted as not given with an explanation of the reason documented in
the nursing progress notes. In this instance, the refusal indicates good judgment by the patient, and the patient should not be
encouraged to take it today.

The nurse is preparing to administer a daily dose of docusate sodium to a patient that will continue taking it after discharge. What
information should the nurse provide to the patient to optimize the outcome of the medication?

a. Take a dose of mineral oil at the same time.


b. Add extra salt to food on at least one meal tray.
c. Ensure a dietary intake of 10 g of fiber each day.
d. Take each dose with a full glass of water or other liquid.

take each dose with a full glass of water or other liquid

Docusate lowers the surface tension of stool, permitting water and fats to penetrate and soften the stool for easier passage. The
patient should take the dose with a full glass of water and should increase overall fluid intake, if able, to enhance effectiveness of
the medication. Dietary fiber intake should be a minimum of 20 g daily to prevent constipation. Mineral oil and extra salt are not
recommended.

The nurse is administering a cathartic agent to a patient with renal insufficiency. Which order will the nurse question?

a. Bisacodyl
b. Lubiprostone
c. Cascara sagrada
d. Magnesium hydroxide

magnesium hydroxide

Milk of Magnesia may cause hypermagnesemia in patients with renal insufficiency. The nurse should question this order with the
health care provider. Bisacodyl, lubiprostone, and cascara sagrada are safe to use in patients with renal insufficiency as long as the
patient is not currently dehydrated.

____ 1. The nurse is collecting data from a newly admitted patient. Which finding should the nurse identify as a risk factor for
constipation?
a. The patient does not like milk or milk products.
b. The patient has had hemorrhoids for the past 5 years.
c. The patient had part of the stomach removed 10 years ago because of ulcers.
d. The patient has a history of breast cancer treated with chemotherapy 3 years ago.

. B The patient has had hemorrhoids for the past 5 years

____ 2. The nurse is contributing to a patient's plan of care. For which patient would the nursing diagnosis of Risk for Constipation
be most appropriate?
a. A 37-year-old taking NSAIDs for bursitis
b. A 59-year-old taking narcotics for chronic pain control
c. A 74-year-old taking antibiotics for a urinary tract infection
d. A 67-year-old taking anticoagulant therapy for a history of deep vein thrombosis

B.59-year-old taking narcotics for chronic pain control

____ 3. The nurse is caring for a patient who reports feeling constipated, yet passes frequent small liquid stools. Which action
should the nurse take?
a. Check the patient for a fecal impaction.
b. Administer an antidiarrheal medication.
c. Explain that liquid stools indicate diarrhea.
d. Check the abdomen for rebound tenderness.

A. Check the patient for a fecal impaction.

___ 4. The nurse notes that a patient with a history of a myocardial infarction is straining during defecation. Which response by the
nurse is best?
a. "Be careful, you might get a headache when you push so hard."
b. "It is important that you not strain because it could cause damage to your heart."
c. "Your blood pressure gets very low when you strain like that and you could faint."
d. "Chronic constipation often causes a dilated colon, so it is good that you are staying empty."

B. "It is important that you not strain because it could cause damage to your heart."

____ 5. A patient with a colostomy says, "My pouch blows up like a balloon when I pass gas." What is an appropriate response by
the nurse?
a. "Make a tiny pinhole in the top of the pouch to let air out."
b. "Empty the gas like you would if the pouch was full of stool."
c. "Peel back a tiny corner of the skin barrier to allow gas to escape."
d. "Remove the pouch and put on a new one when it gets too full of gas."

B. Empty the gas like you would if the pouch was full of stool.

____ 6. The nurse is reinforcing teaching provided to a patient with acute diarrhea. Which statement indicates the patient
understands the most common cause for this health problem?
a. "Excessive fluid intake."
b. "Excessive fiber in the diet."
c. "Viral or bacterial infection."
d. "Inflammatory bowel disease."

C. Excessive fiber in the diet."

____ 7. The nurse is providing discharge teaching to a patient with diarrhea. Which patient statement indicates that teaching has
been effective?
a. "It is important that I increase fluid intake to prevent dehydration."
b. "I am at increased risk for a ruptured bowel, so I must remain on bedrest."
c. "I should tell future health-care workers that I've been diagnosed with obstipation."
d. "My risk for a urinary tract infection is very high, so I should call the doctor if I have a pain."

A. a. "It is important that I increase fluid intake to prevent dehydration."

____ 8. The nurse is caring for a patient who has diarrhea. Which nursing action is the highest priority?
a. Provide perineal skin care.
b. Auscultate the abdomen daily.
c. Encourage oral fluid replacement.
d. Provide analgesics for abdominal pain.

____ 9. The nurse is reinforcing patient teaching on the best way to prevent transmission of infectious diarrhea. Which patient
statement indicates correct understanding of the teaching?
a. Wear a mask and gown.
b. Avoid sharing eating utensils.
c. Keep the perineal area clean and dry.
d. Wash hands frequently and after toileting.

____ 10. The nurse is contributing to the plan of care for a patient with gluten enteropathy (celiac disease). What should the nurse
recommend be eliminated from the diet of the patient?
a. Red meats
b. Milk and milk products
c. Fresh fruits and vegetables
d. Wheat, rye, oats, and barley

____ 11. The nurse is collecting data from a patient who is reporting abdominal pain. Which symptom suggests that the patient is
experiencing appendicitis?
a. Suprapubic pain
b. Midepigastric pain
c. Substernal pain that radiates to the back
d. Pain in the right lower abdominal quadrant

____ 12. The nurse suspects appendicitis in a patient complaining of abdominal pain. Which assessment finding should cause the
nurse to notify the physician?
a. The patient burps after drinking a glass of water.
b. Tympanic, hollow sounds are heard on percussion.
c. Bowel sounds are hyperactive in the upper quadrants.
d. Palpation of the abdomen is positive for rebound tenderness.

____ 13. The nurse is caring for a patient with an inflamed appendix. Which complication is most likely to occur if the appendix
ruptures?
a. Colitis
b. Enteritis
c. Hepatitis

____ 14. The nurse is monitoring a patient recovering from an emergency appendectomy. Which finding should be reported to the
physician immediately?
a. Pain at the operative site
b. Absence of bowel sounds
c. Abdomen rigid on palpation
d. 3-centimeter spot of bloody drainage on dressing

____ 15. A patient is to be started on clear liquids after an appendectomy. Which food should the nurse identify as being a clear
liquid?
a. "Oatmeal."
b. "Ice cream."
c. "Cranberry juice."
d. "Graham crackers."

____ 16. The nurse is caring for a patient who is being screened for diverticulosis. Which patient statement indicates understanding
of conditions that predispose to diverticulosis?
a. "Colon cancer."
b. "Chronic diarrhea."
c. "Chronic constipation."
d. "Diet high in red meats."

____ 17. A patient asks what causes diverticulitis. How should the nurse respond?
a. "The lining of your colon is irritated and inflamed."
b. "You have little pouches in your colon that are inflamed."
c. "You have little outpouchings that occur in weak areas of the colon."
d. "The visceral and parietal membranes in your abdomen are inflamed."

____ 18. The nurse is teaching a patient with diverticulosis how to avoid complications. Which patient statement indicates that
teaching has been effective?
a. "I will avoid milk and milk products."
b. "I should avoid very hot and spicy foods."
c. "I will increase fluids and fiber in my diet."
d. "I should cook vegetables thoroughly before eating."

____ 19. The nurse is caring for a patient with an exacerbation of Crohn's disease. Which nursing action is most important to
recommend for inclusion in the patient's plan of care?
a. Encourage oral fluids.
b. Encourage frequent ambulation.
c. Administer anti-gas agents as ordered.
d. Apply protective ointment to perianal skin.

____ 20. A patient scheduled for an ileostomy for Crohn's disease asks the nurse to explain the procedure. What should the nurse
respond?
a. "You will have a loop of colon brought out onto your abdomen."
b. "Your ileum will be anastomosed to your rectum, so your stools will be watery."
c. "Your ileum will be removed, and the end of your jejunum will be made into a stoma."
d. "Your colon will be

____ 21. A patient with a new ileostomy asks if a bag needs to be worn on the abdomen. What is the most appropriate response by
the nurse?
a. "Your stool will be liquid, so you will always need a bag."
b. "Your stool will be mushy, and you will need a bag most of the time."
c. "You will be taught to irrigate your stoma to eliminate the need for a bag."
d. "Your stool will be formed, and you may be able to regulate your bowel movements so that a bag will be optional."

____ 22. The nurse is caring for a patient recovering from ileostomy surgery. What should have the highest priority when caring for
the patient after surgery?
a. Food intake
b. Participation in stoma care
c. Stoma condition every 8 hours
d. Bowel sounds every 4 hours for 24 hours

____ 23. The nurse is monitoring a patient and finds a bulging area in the patient's groin. Which additional finding should cause the
nurse the most concern?
a. The bulging disappears at times.
b. The white blood cell count is 10,000/mm3.
c. The patient develops pain at the site and vomiting.
d. The bulging occurs when the patient coughs or strains.

____ 24. The nurse is caring for a patient with an absorption disorder. What term should the nurse use to document fat in the
patient's stool?
a. Oleorrhea
b. Steatorrhea
c. Lactorrhea
d. Lipidorrhea

____ 25. A patient with a bowel obstruction asks for the term that describes telescoping of the bowel. Which should the nurse
respond to this patient?
a. "Ileus."
b. "Volvulus."
c. "Adhesions."
d. "Intussusception."
D

__ 26. The nurse is caring for a patient admitted with a possible bowel obstruction. Which patient symptom should cause the nurse
the most concern?
a. Flank pain
b. Fecal vomiting
c. Watery diarrhea
d. Occult blood in the stool

____ 27. While receiving report from the previous shift, the nurse is informed that a nasogastric tube was placed in a patient who
has a bowel obstruction. For which reason should the nurse realize the tube was inserted?
a. To feed the patient
b. To relieve distention
c. To administer medications
d. To prevent another obstruction

____ 28. On admission, a patient with gastrointestinal bleeding had vital signs of a blood pressure of 140/80 mm Hg, pulse 72
beats/minute, respirations 14 breaths/minute, and temperature 98.8°F (37.1°C). What finding should be reported to the registered
nurse (RN) or physician immediately?
a. Pulse 78 beats/minute
b. bdominal

c. Occult blood in the stool


d. Blood pressure 104/68 mm Hg

____ 29. The nurse is caring for a patient who has an ileostomy and feels crampy. The nurse notes that the stoma has become
edematous and pale and suspects a blockage. What action should the nurse take?
a. Administer a laxative such as milk of magnesia.
b. Have the patient drink 2 to 3 L of water or other liquid.
c. Administer a 1000-mL warm tap water enema through the stoma.
d. Have the patient get into a tub full of warm water and drink warm liquids.

____ 30. The nurse is collecting data from a patient with a stoma. What should the nurse document for a health stoma?
a. Gray and dry
b. Black and dry
c. Bluish and wet
d. Pink and moist

____ 31. The nurse is contributing to the plan of care for patient with an ostomy. Why should the nurse recommend the use of a skin
barrier product under the ostomy appliance?
a. To keep stool from irritating the skin
b. To ease removal of the pouch for changing
c. To prevent the bag from sticking too tightly to the skin
d. To prevent stool from coming in contact with the stoma

____ 32. The nurse is evaluating a patient's ability to change an ostomy appliance. Which observation indicates that the patient can
safely provide self-ostomy care?
a. Stoma measured prior to applying new appliance
b. Skin barrier applied tight to the base of the stoma
c. Skin barrier cut to the same size as previous barrier
d. Lotion applied to skin before application of skin barrier

____ 33. The spouse of a patient with an ascending ostomy asks if the patient will always have to wear a pouch. What response
should the nurse make?
a. "A bag will be needed all of the time."
b. "A bag will be needed only during the night."
c. "A bag will be needed only to protect the stoma."
d. "No, a bag will not be needed after discharge from the hospital."

____ 34. A patient is experiencing melena. What does this observation indicate to the nurse?
a. The patient has a ruptured diverticulum
b. The patient has ingested a large volume of red meat
c. Blood has begun to seep into the stomach over the last 3 hours from esophageal varices
d. Blood has been in the gastrointestinal tract for more than 8 hours after being in contact with hydrochloric acid

____ 35. The nurse is reviewing the process of digestion with a patient diagnosed with malabsorption syndrome. How many mL of
fluid should the nurse instruct that is absorbed through the intestinal mucosa into the portal bloodstream?
a. 1000
b. 2000
c. 4000

d. 8000

____ 36. The nurse is reinforcing teaching provided to a patient about appropriate diet modifications to help prevent exacerbations
of inflammatory bowel disease. Which patient statements indicate that teaching has been effective? (Select all that apply.)
a. "I should avoid caffeine and spicy fiber foods."
b. "I should avoid concentrated sweets and starches."
c. "It is important to eat more whole grains and bran."
d. "High-fiber foods should not be included in my diet."
e. "I should increase my intake of fresh fruits and vegetables."
f. "Milk and other dairy products should be limited in my diet."

A, D, F

____ 37. The nurse is participating in a community health fair program focusing on risk factors for cancer. Which should be included
as increasing the risk for colon cancer? (Select all that apply.)
a. Low-fat diet
b. Low-fiber diet
c. Low-sodium diet
d. History of rectal polyps
e. History of ulcerative colitis
f. Family history of breast cancer

B, D, E

____ 38. The nurse reinforces teaching provided to a patient with constipation and straining who is experiencing abdominal
distention and intestinal rumbling. What should be included in the teaching? (Select all that apply.)
a. Set a time for defecation every day.
b. Increase the intake of foods containing vitamin K.
c. Increase intake of fiber, especially bran, in the diet.
d. Sit on the toilet with feet planted firmly on the floor.
e. Drink water each morning and about 2 to 3 L throughout the day.
f. Use enemas and rectal suppositories if constipation persists after 2 days.

A, C, E

____ 39. The nurse is reinforcing teaching provided to a patient who is being discharged with a new colostomy. Which comments by
the patient indicate understanding of the discharge teaching? (Select all that apply.)
a. "I will empty the pouch when it is less than half full."
b. "I can spray deodorant into the pouch after I clean it."
c. "I will not be concerned if there is no stool for several days."
d. "I'm so glad I can eat all the foods I like now, including hot dogs."
e. "I always check the seal and tape around the stoma after I shower."
f. "I should change the pouch each morning and evening to prevent infection."

A, B, E
____ 40. The nurse provides teaching to a patient prescribed budesonide (Entocort EC) for Crohn's disease inflammation. Which
patient statements indicate that more teaching is necessary? (Select all that apply.)
a. "I should avoid grapefruit juice."
b. "I must avoid the sun while taking this drug."
c. "I should swallow the pill whole, not crushed."
d. "I will take the pill each evening before going to bed."
e. "I can just stop taking the medication once I feel better."
f. "I might experience mood swings or weight gain on this medication."

B, D, E

____ 41. The nurse is contributing to a patient's plan of care. Which foods should the nurse recommend to be avoided or used with
caution to reduce the possibility of ileostomy blockage? (Select all that apply.)
a. Celery
b. Apples
c. Potatoes
d. Dried fruits

A,B,D,E

____ 42. A patient with fecal incontinence has an excoriated perianal region. Which interventions should be discussed with the RN?
(Select all that apply.)
a. Stool culture
b. Antibiotic therapy
c. Protective barrier cream
d. Baby powder to peri area
e. A low-pressure rectal tube
f. Nasogastric (NG) tube to suction

A, C, E

____ 43. A patient comes into the client after experiencing diarrhea with five liquid stools in the past 24 hours. Which additional
patient symptoms should cause the nurse concern? (Select all that apply.)
a. Fever
b. Blood in the stool
c. Severe abdominal cramping
d. Blood pressure 138/72 mm Hg
e. Oral intake of 3 L of fluid in 24 hours
f. Weight loss of 1 pound in the past week

A, B, C

____ 44. During a health history, the nurse learns that a patient uses laxatives every day to ensure a bowel movement. What should
the nurse expect to be prescribed for this patient? (Select all that apply.)
a. Daily enema
b. Psyllium (Metamucil)
c. Daily rectal suppository
d. Docusate sodium (Colace)
e. Methylnaltrexone (Relistor)

B, D

____ 45. The nurse is assisting to prepare dietary teaching for a patient with diverticulosis. Which food items should the nurse
suggest be added to this patient's teaching plan? (Select all that apply.)
a. Peas
b. Salad
c. Cheese
d. Prunes
e. Raisins

A, B, D, E

____ 46. The nurse is teaching a patient newly diagnosed with ulcerative colitis about triggers for exacerbation of the disease. What
should the nurse urge the patient to do to prevent a future exacerbation? (Select all that apply.)
a. Do not use tobacco
b. Reduce exposure to stress
c. Restrict fluids to 2 liters per day
d. Read food labels to avoid food additives
e. Avoid ingesting foods sprayed with pesticides

A, B, D, E

____ 47. A patient with Crohn's disease is scheduled for an ileoanal pouch. What should the nurse include when teaching the
patient about this surgery? (Select all that apply.)
a. Stool will pass through the anus.
b. A temporary ileostomy is needed.
c. The stool is hard and brown in color.
d. An ostomy pouch will need to be worn.
e. Several bowel movements occur per day.

A, B, E

____ 48. A patient with irritable bowel syndrome is being started on the FODMAP diet. What foods should the nurse instruct the
patient to avoid when following this diet? (Select all that apply.)
a. Milk
b. Pears
c. Apples
d. Broccoli
e. Brussels sprouts

A, B, C, E

____ 1. A patient with possible viral hepatitis reports recent intake of raw shellfish. Which type of hepatitis should the nurse consider
the patient is experiencing?
a. Hepatitis A virus
b. Hepatitis B virus
c. Hepatitis C virus
d. Hepatitis D virus

b. Hepatitis B virus
c. Hepatitis C virus
d. Hepatitis D virus

____ 2. The employee health nurse is preparing vaccines to administer to patient care staff to permanently protect them from
hepatitis. For which types of hepatitis does the nurse have vaccines?
a. HAV
b. HBV
c. HCV
d. Both HAV and HBV

____ 3. A patient with hepatic encephalopathy is required to consume 50 grams of protein each day. Which item should be provided
to the patient for a mid-afternoon snack?
a. Apple
b. Crackers
c. Peanut butter
d. Whole grain bread

____ 4. The nurse is caring for a patient after a liver transplant. Which symptom should the nurse report immediately as a possible
indication of rejection of the liver?
a. Pulse rate of 80 beats per minute
b. Prothrombin time (PT) of 14 seconds
c. Decreased alanine aminotransferase (ALT)
d. A temperature greater than 101°F (38.3°C)

____ 5. The nurse is reinforcing teaching provided to a patient with chronic liver failure. What should the patient be instructed to help
prevent injury?
a. "Drink plenty of clear fluids."
b. "Brush your teeth with a soft-bristled brush."
c. "Be sure to get 20 minutes of exercise daily."
d. "Take an aspirin a day to prevent heart complications."

____ 6. A patient with liver failure and esophageal varices is prescribed to receive vasopressin. What should the nurse realize is the
purpose for this medication?
a. To promote portal circulation
b. To reduce ammonia buildup and encephalopathy
c. To constrict vessels causing bleeding in esophageal varices
d. To maintain blood pressure in a patient with hypotension related to bleeding varices

____ 7. The nurse is caring for a patient with chronic liver failure. Which laboratory value should the nurse expect as a late sign of
liver failure?
a. Low serum albumin
b. Low serum bilirubin
c. Low serum ammonia
d. Low serum aspartate aminotransferase (AST)

____ 8. The nurse is caring for a patient with hepatic encephalopathy. Which prescribed medication should the nurse question
before providing to this patient?
a. Vitamin K
b. Neomycin sulfate
c. Diazepam (Valium)
d. Lactulose (Cephulac)

____ 9. The nurse is caring for a patient with esophageal varices. Which symptom should alert the nurse to possible bleeding?
a. Asterixis
b. Dark amber urine
c. Hard formed stool
d. Blood-streaked emesis

____ 10. The nurse is reinforcing teaching provided to a patient with esophageal varices. Which activity should the patient be taught
to avoid?
a. Lifting heavy objects
b. Participating in aerobic activities
c. Eating concentrated carbohydrates
d. Rising suddenly from a reclining position

____ 11. The nurse is collecting data from a patient with liver failure to detect encephalopathy. What instructions should the nurse
give to the patient to collect this data?
a. "Stand with your eyes closed."
b. "Hold out your arms and hands."
c. "Kneel on your hands and knees."
d. "Bear down as though you were having a bowel movement."

____ 12. The nurse is collecting data from a patient with acute pancreatitis. Which symptoms should the nurse anticipate?
a. Low abdominal pain, bradycardia, and confusion
b. Shortness of breath, hypotension, and restlessness
c. Fever, tachycardia, right upper quadrant pain, and jaundice
d. Abdominal distention, respiratory distress, and mid-epigastric pain

D
____ 13. The nurse is collecting data for a patient with acute pancreatitis. Which laboratory test result should the nurse expect?
a. Decreased serum lipase
b. Elevated serum amylase
c. Elevated serum albumin
d. Decreased serum ammonia

____ 14. The nurse is reinforcing teaching provided to a patient with a history of acute pancreatitis. Which item should the patient be
instructed to avoid?
a. High-sodium foods
b. Alcoholic beverages
c. Carbonated beverages
d. Foods with preservatives

____ 15. The nurse is caring for a patient with acute pancreatitis who is vomiting. What should the nurse frequently assess in this
patient?
a. Skin color and pain
b. Vital signs and urinary output
c. Bowel sounds and body weight
d. Ability to move lower extremities

____ 16. A patient with acute pancreatitis is experiencing severe pain. What position should the nurse encourage the patient to
assume?
a. Semi-Fowler's position
b. Prone with a pillow under the abdomen
c. Supine with legs elevated and head on a small pillow
d. Sitting in a chair leaning forward with a pillow for back support

____ 17. The nurse is reinforcing discharge teaching about recurrence of pancreatitis to a patient with chronic pancreatitis. What
information should the nurse include?
a. Periodic epigastric pain is a normal occurrence.
b. Report anorexia, hyperglycemia, or weight loss.
c. Recurrence of pancreatitis is unlikely to happen.
d. Report jaundice, flatulence, or amber-colored urine.

____ 18. The nurse is caring for a patient after surgery to drain a pancreatic abscess. Which action should the

nurse take to monitor for complications?


a. Document output.
b. Monitor blood glucose.
c. Monitor for hyperproteinemia.
d. Review serum potassium levels.

____ 19. The nurse is reinforcing teaching provided to a patient with gallstones. What substance should the nurse instruct that
makes up most gallstones?
a. Sodium
b. Calcium
c. Cholesterol
d. Phosphorus

____ 20. A patient with gallstones asks why jaundice has developed. What should the nurse explain as the most likely cause for the
patient's jaundice?
a. Hepatitis
b. Cirrhosis
c. Hemolysis
d. Bile duct obstruction

____ 21. A patient with cholelithiasis is having clay-colored stools. What should the nurse realize as the most common cause of
clay-colored stools?
a. Retrograde bile flow into the liver
b. Accumulation of bile salts in the skin
c. Cirrhosis from chronic liver irritation
d. A gallstone lodged in the common bile duct

____ 22. The nurse is contributing to the teaching plan for a patient recovering from a cholecystectomy. Which dietary modification
should the nurse recommend for the first few weeks after surgery?
a. Decrease intake of fresh fruits and vegetables to minimize pressure on the small intestine.
b. Consume at least four servings of meat, cheese, and peanut butter daily to boost protein intake and aid healing.
c. Distribute fat intake in small portions throughout the day to prevent excessive fat in the intestine at any one time.
d. Take pancreatic enzymes with meals to replace enzymes that would normally have been secreted before the cholecystectomy.

___ 23. A patient with cholecystitis is prescribed promethazine (Phenergan) for nausea. Which adverse effect of the medication
should the nurse instruct the patient to report?
a. Diarrhea
b. Insomnia
c. Dry mouth
d. Urine retention

____ 24. The nurse is caring for a patient with chronic liver failure. Which medication order should the nurse question?
a. Lactulose
b. Neomycin
c. Multivitamins
d. Acetaminophen

____ 25. A patient recovering from a cholecystectomy earlier in the day is reluctant to deep breathe and cough. What intervention
should the nurse use to assist the patient to cough and breathe?
a. Remind the patient to deep breathe and cough every hour.
b. Teach the patient to use relaxation and distraction techniques.
c. Medicate the patient for pain, and assist to splint the abdomen.
d. Reinforce the importance of the deep breathing and coughing activities.

____ 26. The nurse is identifying care to delegate to unlicensed assistive personnel. Which actions could be safely delegated in the
care of a patient with fulminant liver failure?
a. Evaluating the patient's mental status
b. Assisting with bathing and positioning
c. Assessing the stool and urine for blood
d. Monitoring laboratory studies for abnormal values

____ 27. A patient with biliary colic is prescribed an anticholinergic medication to help treat biliary colic. For which medical diagnosis
should the nurse question the administration of this medication?
a. Asthma
b. Psoriasis
c. Diabetes mellitus
d. Prostatic hypertrophy

D
____ 28. The nurse is providing a patient with cholelithiasis the medication ursodiol (Actigall). What should the nurse instruct the
patient about this medication?
a. This medication is used prior to having surgery.
b. This medication works best with a high-fat diet.
c. This medication may take a few months to work.
d. This medication makes sure the stones never return.

____ 29. A patient with chronic liver disease is prescribed dexlansoprazole (Kapidex). What should the nurse instruct the patient
about this medication?
a. Take the entire dose of medication whole.
b. Crush the medication and sprinkle in water.
c. Take this medication with a full glass of milk.
d. Take half the medication with breakfast and the other half with dinner

____ 30. A patient with liver failure takes acetaminophen (Tylenol) 650 mg tablets by mouth for severe arthritis pain. How many
tablets should the nurse instruct that the patient can safely take in one 24 hour period?
a. 2
b. 3
c. 4
d. 5

____ 31. The nurse is caring for a patient recovering from an incisional cholecystectomy. Which activities should the nurse identify
as having the highest priority for this patient? (Select all that apply.)
a. Managing pain
b. Performing leg exercises
c. Coughing and deep breathing
d. Ambulating early and frequently
e. Choosing low-fat foods from the menu
f. Encouraging use of an incentive spirometer

A, C, D, F

____ 32. The nurse is reinforcing teaching provided to a patient recovering from an acute attack of cholecystitis. Which foods should
the nurse caution the patient to avoid? (Select all that apply.)
a. Rice
b. Eggs
c. Cheese
d. Lean meats
e. Fresh fruits

B, C

___ 33. The nurse is providing education to a patient recovering from a recent cholecystectomy. What should the nurse include in
the teaching? (Select all that apply.)
a. "Fat should be less than 20% of total diet."
b. "Raw fruits and vegetables should be avoided."
c. "It is important to increase the protein intake in your diet."
d. "If you are overweight, it is suggested that you lose weight."
e. "There are no dietary restrictions once you leave the hospital."
f. "Fat is introduced slowly and adjusted according to individual tolerance."

C, D, F

____ 34. The nurse is collecting data for a patient with acute liver failure. Which laboratory test findings should the nurse recognize
as supporting this diagnosis? (Select all that apply.)
a. Elevated platelet count
b. Elevated prothrombin time
c. Elevated serum bilirubin level
d. Elevated serum potassium level
e. Elevated alanine aminotransferase level (ALT)
f. Elevated aspartate aminotransferase level (AST)
B, C, E, F

____ 35. A patient with pancreatitis is receiving care to address the nursing diagnosis Imbalanced Nutrition: Less than required
related to pain, NPO, and nasogastric suction. After 10 days of treatment, which findings should indicate to the nurse that the
treatment plan has been effective? (Select all that apply.)
a. The patient reports pain relief.
b. The serum sodium is 130 mEq/L.
c. The patient's albumin level is 3.8 g/L.
d. The serum potassium level is 3.7 mEq/L.
e. The patient has mild diarrhea and steatorrhea.
f. The patient has returned to baseline body weight.

C, F

____ 36. A patient has the nursing diagnosis of Deficient Fluid Volume related to anorexia, nausea, vomiting, and excessive T-tube
drainage related to cholecystitis. Which interventions should the nurse recommend be included in the plan of care? (Select all that
apply.)
a. Monitor skin turgor.
b. Administer antiemetics as ordered.
c. Clamp T-tube for 2 hours each shift.
d. Monitor daily weight and intake and output.
e. Encourage use of incentive spirometer every hour while awake.
f. Contact the physician if T-tube drainage is greater than 150 mL within 24 hours of surgery.

A, B, D

____ 37. The nurse is caring for a patient diagnosed with chronic hepatitis B. Which medications should the nurse anticipate being
prescribed for this patient? (Select all that apply.)
a. Interferon alpha-2a
b. Ribavirin (Rebetol)
c. Adefovir (Hepsera)
d. Lamivudine (Epivir)
e. Peginterferon alpha-2b

C, D

____ 38. A patient recovering from hepatitis is concerned about liver damage from the infection. What should the nurse instruct the
patient to do to prevent long-term liver damage? (Select all that apply.)
a. Get adequate rest.
b. Ingest nutritious foods.
c. Abstain from all alcohol.
d. Restrict physical activity.
e. Limit the intake of dairy products.

A, B, C

39. The nurse is instructing the mother of an adolescent with hepatitis on ways to prevent the spread of infection in the home. What
should the nurse include in this mother's teaching? (Select all that apply.)
a. Use bar soap.
b. Wear rubber gloves when handling the patient's used laundry.
c. Wash contaminated linens separately from other family linens.
d. Identify a separate bedroom and bathroom for the patient to use.
e. Wash gloves with 10% bleach solution after use for cleaning the bathroom.

B, C, D, E

____ 40. While collecting data, the nurse becomes concerned that a patient is at risk for developing liver cancer. What information
did the nurse use to come to this conclusion? (Select all that apply.)
a. Lives in an urban community
b. Ingests four six-packs of beer each day
c. Smokes two packs of cigarettes each day
d. Has a history of chronic hepatitis B infection
e. Employed as a remote computer operator

B, C, D

You might also like